LSAT and Law School Admissions Forum

Get expert LSAT preparation and law school admissions advice from PowerScore Test Preparation.

 Administrator
PowerScore Staff
  • PowerScore Staff
  • Posts: 8919
  • Joined: Feb 02, 2011
|
#31750
Please post below with any questions!
 roselsat123
  • Posts: 3
  • Joined: Jan 17, 2017
|
#32158
Can you explain why A is the correct answer?
 Kristina Moen
PowerScore Staff
  • PowerScore Staff
  • Posts: 230
  • Joined: Nov 17, 2016
|
#32185
Rose,

Welcome to the Forum! We can probably help you more if we know more about what you're thinking. Was there an answer choice that you liked better? Why?

For now, let's deconstruct this question. The stimulus contains conditional reasoning (evidence by Sufficient Condition Indicator "if"), so I start off diagramming right away!

Purpose of laws is to contribute to people's happiness -> We have a basis for criticizing current laws and proposing new laws.
You might also shorthand this as PH -> CL.

Next sentence: If no PH, then we have no basis for criticizing them.

STOP RIGHT THERE! The author committed a logical flaw. If the sufficient condition is not met, that does NOT mean that the necessary condition doesn't happen. It's a Mistaken Negation.

Then the author goes to conclude something based on that logical flaw.

The questions type is Flaw in the Reasoning. I already know it's a flaw in conditional reasoning. The author concluded from PH -> CL that if no PH, then no CL. Then he went to basis his conclusion off of that logical flaw.

Answer choice (A) describes a flaw in conditional reasoning.
 avengingangel
  • Posts: 275
  • Joined: Jun 14, 2016
|
#47942
If we're agreeing the second sentence isn't the arguments overall conclusion, then it's the arguments intermediary conclusion, which means it's a premise that the argument uses to support it's overall conclusion: "existing laws acquire legitimacy simply because they are the laws." But we must take premises as facts. (Which is why I didn't choose A, because I accepted it as true, and not a case of the MN flaw in reasoning) So, I guess my question is, as a rule, can we cast doubt on an argument's premise if it's intermediary conclusion?? Thanks!
 Sky Brooks
PowerScore Staff
  • PowerScore Staff
  • Posts: 18
  • Joined: Jul 14, 2018
|
#48890
Hi Avengingangel,

In a question where you are asked to identify the flaw in the argument, you must review the argument and see where it uses invalid reasoning. Despite whether parts of the argument are premises or not (and yes, premises are to be treated as being true) the test taker must identify the flaw.

In this question, the author committed a logical flaw. If the sufficient condition is not met, that does NOT mean that the necessary condition doesn't happen. It's a Mistaken Negation. Even if the premises are treated as being true, they do not add up to a logical argument, they are logically flawed.

Then the author goes to conclude something based on that logical flaw.

The questions type is Flaw in the Reasoning. The author concluded from PH -> CL that if no PH, then no CL. Then he went on to base his conclusion off of that logical flaw.

Answer A directly references that flaw and is the correct answer.

I hope this helps,
Sky
 Tajadas
  • Posts: 63
  • Joined: Apr 11, 2020
|
#81811
I recognize the mistaken negation in the stimulus but I don't understand how that corresponds to answer A. A what point does the sufficient condition become the necessary condition, as A says?
User avatar
 KelseyWoods
PowerScore Staff
  • PowerScore Staff
  • Posts: 1079
  • Joined: Jun 26, 2013
|
#81954
Hi Tajadas!

The answer choice "takes (or mistakes) a sufficient condition for a necessary condition" is actually a really common way that the LSAT uses to describe a Mistaken Negation in Flaw answer choices. I'd recommend that you study and memorize the ways that the LSAT describes common flaws in answer choices because they reuse a lot of the same language and it makes your task of identifying answer choices that describe the flaws that much easier and faster!

As for why that particular wording describes a Mistaken Negation, I recommend that you check out this other post I wrote in response to a student where I go into an in depth analysis of conditional reasoning flaw answer choices: https://forum.powerscore.com/lsat/viewt ... 597#p74597

Hope this helps!

Best,
Kelsey
User avatar
 queenbee
  • Posts: 75
  • Joined: Sep 18, 2022
|
#98088
Hi Kelsey
I am trying to apply the logic to this question but I am missing something:

(sufficient) If (Purpose of the Law is to promote happiness) --> (necessary) we have a basis to criticize
Contra
(sufficient) If NOT (we have a basis to criticize) --> (necessary) NOT (Purpose of the Law is to promote happiness)

Answer A: It takes a sufficient condition to be a necessary condition

stimulus:
(sufficient) if NOT (Purpose of the Law is to promote happiness) --> (necessary) NOT(we have a basis to criticize)

Im just not following where it is taking a sufficient condition to be a necessary condition.
Would you kindly help me out with this?
Thank you
 Rachael Wilkenfeld
PowerScore Staff
  • PowerScore Staff
  • Posts: 1358
  • Joined: Dec 15, 2011
|
#98521
Hi queenbee,

It seems like you have a strong understanding of the conditionals in the stimulus. Let's dive a bit into conditionals to see why the description of the error is correct.

Here they jump from P :arrow: C to P :arrow: C. That looks an awful lot like a mistaken negation. We negated both terms, but did not flip them.

What would have been valid reasoning? Well, it would have been valid to go from a conditional that says C :arrow: P to P :arrow: C. That would be a valid contrapositive. So the error in reasoning is that they switched up the sufficient and the necessary conditions. The flip of the first sentence would be a valid conditional relationship.

One thing that is helpful to keep in mind---a mistaken reversal and a mistaken negation are logically equivalent. One is the contrapositive of the other. So if you recognize that you have a mistaken negation, but the description is a mistaken reversal, that's ok. You've found the correct answer choice.

Hope that helps!
User avatar
 queenbee
  • Posts: 75
  • Joined: Sep 18, 2022
|
#98986
Yes it does...thank you!

Get the most out of your LSAT Prep Plus subscription.

Analyze and track your performance with our Testing and Analytics Package.